Locally finite collection problem

  • Thread starter Thread starter radou
  • Start date Start date
  • Tags Tags
    Finite
Click For Summary
A point-finite open covering of R is defined using the collection A, which consists of intervals that escape to both negative and positive infinity, as well as a shrinking interval around zero. Each point in R is contained in only finitely many elements of A, confirming it is point-finite. However, any neighborhood around zero intersects A in infinitely many elements, demonstrating that A is not locally finite. The discussion concludes with a request for hints on a more complex exercise involving a non-locally finite collection whose closures are locally finite. This example effectively illustrates the concepts of point-finite and locally finite collections in topology.
radou
Homework Helper
Messages
3,148
Reaction score
8

Homework Statement



I'm not especially good at creating examples, so I'd like to check this one.

One needs to find a point-finite open covering of R which is not locally finite. (A collection is point-finite if each point of R lies in only finitely many elements of that collection)

The Attempt at a Solution



Let's define the collection A:

A = {<-n, 3-n> U <n, n + 2> U <0, 1/n> : n is in N}

Now, let's check A is point-finite. Let x be in R. Wherever x is, there exists some n such that x is not in the upper union of intervals indexed by n. (We can think of this as the interval <-n, 3-n> "escaping" to -∞, the interval <n, n + 2> "escaping" to +∞, and <0, 1/n> "escaping to 0, btw excuse me for these primitive notions).

Now, to see A is not locally finite, let U be any neighborhood of 0. Then it intersects the collection A in infinitely many elements, because of the interval <0, 1/n> contained in every element of A.

Thanks in advance for any comments.
 
Physics news on Phys.org
This is entirely correct. It is actually a very nice example! Good job!
 
OK, thanks!
 
Btw, the next exercise seems to be a bit more sophisticated; to find a collection of sets A which is not locally finite, but so that the collection consisting of the closures of these sets is locally finite! Any hints on that one?
 
Question: A clock's minute hand has length 4 and its hour hand has length 3. What is the distance between the tips at the moment when it is increasing most rapidly?(Putnam Exam Question) Answer: Making assumption that both the hands moves at constant angular velocities, the answer is ## \sqrt{7} .## But don't you think this assumption is somewhat doubtful and wrong?

Similar threads

Replies
1
Views
2K
Replies
10
Views
2K
  • · Replies 3 ·
Replies
3
Views
909
  • · Replies 1 ·
Replies
1
Views
2K
  • · Replies 2 ·
Replies
2
Views
1K
  • · Replies 14 ·
Replies
14
Views
2K
  • · Replies 3 ·
Replies
3
Views
2K
  • · Replies 1 ·
Replies
1
Views
2K
  • · Replies 3 ·
Replies
3
Views
2K
  • · Replies 6 ·
Replies
6
Views
2K